Difference between revisions of "Aufgaben:Exercise 5.7Z: Matched Filter - All Gaussian"

From LNTwww
m (Text replacement - "[[Stochastische_Signaltheorie/" to "[[Theory_of_Stochastic_Signals/")
 
(6 intermediate revisions by 3 users not shown)
Line 1: Line 1:
  
{{quiz-Header|Buchseite=Stochastische Signaltheorie/Matched-Filter
+
{{quiz-Header|Buchseite=Theory_of_Stochastic_Signals/Matched_Filter
 
}}
 
}}
  
[[File:P_ID578__Sto_Z_5_7.png|right|frame|Vorgegebener Gaußimpuls]]
+
[[File:P_ID578__Sto_Z_5_7.png|right|frame|Given Gaussian pulse]]
Am Eingang eines Empfangsfilters liegt ein von weißem Rauschen mit der Rauschleistungsdichte  $N_0 = 10^{-4}\hspace{0.08cm} \rm V^2 \hspace{-0.1cm}/Hz$  überlagerter Gaußimpuls  $g(t)$  mit der Amplitude  $g_0$  und der äquivalenten Dauer  $\Delta t_g  = 1\hspace{0.08cm} \rm ms$  an:
+
At the input of a reception filter there is
:$$g(t) = g_0  \cdot {\rm{e}}^{ - {\rm{\pi }}\left( {t/\Delta t_g } \right)^2 } .$$
+
*a Gaussian pulse   $g(t)$  with amplitude  $g_0$  and equivalent duration $\Delta t_g  = 1\hspace{0.08cm} \rm ms$,
 +
:$$g(t) = g_0  \cdot {\rm{e}}^{ - {\rm{\pi }}\left( {t/\Delta t_g } \right)^2 } ,$$
 +
*superimposed by white Gaussian noise with power density $N_0 = 10^{-4}\hspace{0.08cm} \rm V^2 \hspace{-0.1cm}/Hz$.
  
Die Impulsenergie beträgt  $E_g = 0.01\hspace{0.08cm} \rm V^2 s$.  Das Empfangsfilter sei ein akausaler Gaußtiefpass mit dem Frequenzgang
+
 
 +
The pulse energy is  $E_g = 0.01\hspace{0.08cm} \rm V^2 s$.  Let the reception filter be an acausal Gaussian low-pass filter with frequency response
 
:$$H_{\rm E} (f) = {\rm{e}}^{ - {\rm{\pi }}\left( {f/\Delta f_{\rm E} } \right)^2 } .$$
 
:$$H_{\rm E} (f) = {\rm{e}}^{ - {\rm{\pi }}\left( {f/\Delta f_{\rm E} } \right)^2 } .$$
  
Die dazugehörige Impulsantwort lautet somit:
+
The associated impulse response is thus:
 
:$$h_{\rm E} (t) = \Delta f_{\rm E}  \cdot {\rm{e}}^{ - {\rm{\pi }}\left( {\Delta f_{\rm E}  \hspace{0.03cm}\cdot \hspace{0.03cm}t} \right)^2 } .$$
 
:$$h_{\rm E} (t) = \Delta f_{\rm E}  \cdot {\rm{e}}^{ - {\rm{\pi }}\left( {\Delta f_{\rm E}  \hspace{0.03cm}\cdot \hspace{0.03cm}t} \right)^2 } .$$
  
Die systemtheoretische Filterbandbreite  $\Delta f_{\rm E}$  soll so gewählt werden, dass der Gaußtiefpass optimal an den Eingangsimpuls  $g(t)$  angepasst ist. Man spricht dann von einem Matched-Filter.
+
The system-theoretical filter bandwidth  $\Delta f_{\rm E}$  should be chosen so that the Gaussian low-pass filter is optimally matched to the input impulse  $g(t)$.  This is then referred to as a:  "matched filter".
 
 
 
 
  
  
Line 21: Line 22:
  
  
''Hinweise:''
+
Notes:  
*Die Aufgabe gehört zum  Kapitel  [[Theory_of_Stochastic_Signals/Matched-Filter|Matched-Filter]].
+
*The exercise belongs to the chapter  [[Theory_of_Stochastic_Signals/Matched_Filter|Matched Filter]].  
+
*Use the following definite integral to solve:
*Benutzen Sie zur Lösung das folgende bestimmte Integral:
 
 
:$$\int_0^\infty  {{\rm{e}}^{ - a^2 x^2 } {\rm{d}}x = \frac{{\sqrt {\rm{\pi }} }}{2a}} .$$
 
:$$\int_0^\infty  {{\rm{e}}^{ - a^2 x^2 } {\rm{d}}x = \frac{{\sqrt {\rm{\pi }} }}{2a}} .$$
  
Line 30: Line 30:
  
  
===Fragebogen===
+
===Questions===
  
 
<quiz display=simple>
 
<quiz display=simple>
{Berechnen Sie die Impulsamplitude.
+
{Calculate the pulse amplitude.
 
|type="{}"}
 
|type="{}"}
 
$g_0 \ = \ $ { 2.659 3% } $\ \rm V$
 
$g_0 \ = \ $ { 2.659 3% } $\ \rm V$
  
  
{Wie groß ist das maximale S/N&ndash;Verhältnis am Filterausgang in&nbsp; $\rm dB$?
+
{What is the maximum S/N ratio at the filter output in&nbsp; $\rm dB$?
 
|type="{}"}
 
|type="{}"}
 
$10 \cdot \lg\ \rho_d(T_\text{D, opt}\hspace{-0.05cm}) \ =  \ $ { 23 3% } $\ \rm dB$
 
$10 \cdot \lg\ \rho_d(T_\text{D, opt}\hspace{-0.05cm}) \ =  \ $ { 23 3% } $\ \rm dB$
  
  
{Bei welcher Filterbandbreite wird dieses S/N&ndash;Verhältnis erreicht?
+
{At what filter bandwidth is this S/N ratio achieved?
 
|type="{}"}
 
|type="{}"}
 
$\Delta f_\text{E, opt}\ =  \ $ { 1 3% } $\ \rm kHz$
 
$\Delta f_\text{E, opt}\ =  \ $ { 1 3% } $\ \rm kHz$
  
  
{Welche der folgenden Aussagen treffen zu, wenn die Filterbandbreite&nbsp; $\Delta f_{\rm E}$&nbsp; kleiner ist als in der Teilaufgabe&nbsp;  '''(3)'''&nbsp; berechnet?
+
{Which of the following statements are true if the filter bandwidth&nbsp; $\Delta f_{\rm E}$&nbsp; is smaller than calculated in subtask&nbsp;  '''(3)'''?&nbsp;  
 
|type="[]"}
 
|type="[]"}
+ Der Nutzabtastwert&nbsp; $d_{\rm S}(T_\text{D, opt}\hspace{-0.05cm})$&nbsp; ist kleiner als bei Anpassung.
+
+ The signal component&nbsp; $d_{\rm S}(T_\text{D, opt}\hspace{-0.05cm})$&nbsp; is smaller than with matching.
- Die Störleistung&nbsp; $\sigma_d^2$&nbsp; ist größer als  bei Anpassung.
+
- The noise power&nbsp; $\sigma_d^2$&nbsp; is larger than with matching.
+ Das S/N&ndash;Verhältnis ist kleiner als in der Teilaufgabe&nbsp; '''(2)'''&nbsp; berechnet.
+
+ The S/N ratio is smaller than calculated in subtask&nbsp; '''(2)'''.&nbsp;
  
  
Line 58: Line 58:
 
</quiz>
 
</quiz>
  
===Musterlösung===
+
===Solution===
 
{{ML-Kopf}}
 
{{ML-Kopf}}
'''(1)'''&nbsp; Für die Energie eines Impulses&nbsp; $g(t)$&nbsp; gilt allgemein bzw. bei diesem Beispiel:
+
'''(1)'''&nbsp; For the energy of an pulse&nbsp; $g(t)$&nbsp; applies in general,&nbsp; or for this example:
 
:$$E_g  = \int_{ - \infty }^{ + \infty } {g^2(t) \hspace{0.1cm}{\rm{d}}t}  = g^2_0  \cdot \int_{ - \infty }^{ + \infty } {{\rm{e}}^{ - 2{\rm{\pi }}\left( {t/\Delta t_g } \right)^2 } \hspace{0.1cm}{\rm{d}}t} .$$
 
:$$E_g  = \int_{ - \infty }^{ + \infty } {g^2(t) \hspace{0.1cm}{\rm{d}}t}  = g^2_0  \cdot \int_{ - \infty }^{ + \infty } {{\rm{e}}^{ - 2{\rm{\pi }}\left( {t/\Delta t_g } \right)^2 } \hspace{0.1cm}{\rm{d}}t} .$$
  
*Diese Gleichung kann wie folgt umgeformt werden:
+
*This equation can be transformed as follows:
 
:$$E_g  = 2 \cdot g_0 ^2  \cdot \int_0^\infty  {{\rm{e}}^{ - \left( {\sqrt {2 \rm{\pi }} /\Delta t_g } \right)^2  \cdot \hspace{0.05cm} t^2 }\hspace{0.1cm} {\rm{d}}t} .$$
 
:$$E_g  = 2 \cdot g_0 ^2  \cdot \int_0^\infty  {{\rm{e}}^{ - \left( {\sqrt {2 \rm{\pi }} /\Delta t_g } \right)^2  \cdot \hspace{0.05cm} t^2 }\hspace{0.1cm} {\rm{d}}t} .$$
  
*Mit&nbsp; $a = (2\pi)^{1/2}\cdot 1/\Delta t_g$&nbsp; und  der angegebenen Formel gilt folgender Zusammenhang:
+
*With &nbsp; $a = (2\pi)^{1/2}\cdot 1/\Delta t_g$ &nbsp; and the formula given,&nbsp; the following relationship holds:
 
:$$E_g  = 2 \cdot g_0 ^2  \cdot \frac{{\sqrt {\rm{\pi }} }}{2a} = \sqrt 2  \cdot g_0 ^2  \cdot \Delta t_g .$$
 
:$$E_g  = 2 \cdot g_0 ^2  \cdot \frac{{\sqrt {\rm{\pi }} }}{2a} = \sqrt 2  \cdot g_0 ^2  \cdot \Delta t_g .$$
  
*Löst man diese Gleichung nach&nbsp; $g_0$&nbsp; auf, so erhält man als Endergebnis:
+
*Solving this equation for&nbsp; $g_0$,&nbsp; the final result is:
 
:$$g_0  = \sqrt {\frac{E_g }{\Delta t_g  \cdot \sqrt 2 }}  = \sqrt {\frac{{0.01\;{\rm{V}}^{\rm{2}} {\rm{s}}}}{{0.001\;{\rm{s}} \cdot 1.414}}} \hspace{0.15cm}\underline { = 2.659\;{\rm{V}}}.$$
 
:$$g_0  = \sqrt {\frac{E_g }{\Delta t_g  \cdot \sqrt 2 }}  = \sqrt {\frac{{0.01\;{\rm{V}}^{\rm{2}} {\rm{s}}}}{{0.001\;{\rm{s}} \cdot 1.414}}} \hspace{0.15cm}\underline { = 2.659\;{\rm{V}}}.$$
  
  
  
'''(2)'''&nbsp; Unter der Voraussetzung eines Matched&ndash;Filters lautet das S/N-Verhältnis am Ausgang:
+
'''(2)'''&nbsp; Assuming a matched filter,&nbsp; the S/N ratio at the output is:
 
:$$\rho _{d} ( {T_{{\rm{D,}}\,{\rm{opt}}} } ) = \frac{2 \cdot E_g }{N_0 } = \frac{{2 \cdot 10^{ - 2} \;{\rm{V}}^{\rm{2}} {\rm{s}}}}{{10^{ - 4} \;{\rm{V}}^{\rm{2}} {\rm{/Hz}}}} = 200.$$
 
:$$\rho _{d} ( {T_{{\rm{D,}}\,{\rm{opt}}} } ) = \frac{2 \cdot E_g }{N_0 } = \frac{{2 \cdot 10^{ - 2} \;{\rm{V}}^{\rm{2}} {\rm{s}}}}{{10^{ - 4} \;{\rm{V}}^{\rm{2}} {\rm{/Hz}}}} = 200.$$
  
*In logarithmischer Darstellung erhält man folgendes Ergebnis:
+
*In logarithmic representation,&nbsp; the following result is obtained:
 
:$$10 \cdot \lg \rho _{d} ( {T_{{\rm{D,}}\,{\rm{opt}}} } ) = 10 \cdot \lg \left( {200} \right) \hspace{0.15cm}\underline {\approx 23\;{\rm{dB}}}.$$
 
:$$10 \cdot \lg \rho _{d} ( {T_{{\rm{D,}}\,{\rm{opt}}} } ) = 10 \cdot \lg \left( {200} \right) \hspace{0.15cm}\underline {\approx 23\;{\rm{dB}}}.$$
  
  
  
'''(3)'''&nbsp; Ein Vergleich zwischen dem Eingangsimpuls und dem Filterfrequenzgang zeigt, dass bei Anpassung&nbsp; $\Delta f_{\rm E} = 1/\Delta t_g$&nbsp; gelten muss:
+
'''(3)'''&nbsp; A comparison between the input pulse and the filter frequency response shows that when&nbsp; $\Delta f_{\rm E} = 1/\Delta t_g$&nbsp; is fitted,&nbsp; it must hold:
 
:$$\Delta f_{{\rm{E,}}\,{\rm{opt}}} \hspace{0.15cm}\underline { = 1\;{\rm{kHz}}}.$$
 
:$$\Delta f_{{\rm{E,}}\,{\rm{opt}}} \hspace{0.15cm}\underline { = 1\;{\rm{kHz}}}.$$
  
  
  
'''(4)'''&nbsp; Richtig sind <u>die Lösungsvorschläge 1 und 3</u>:
+
'''(4)'''&nbsp; <u>Solutions 1 and 3</u>&nbsp; are correct:
*Eine kleinere Filterbandbreite ist günstig bezüglich Störungen,  
+
*A smaller filter bandwidth is favorable with respect to interference,  
*jedoch ungünstig hinsichtlich des Nutzsignals.  
+
*but unfavorable with respect to the useful signal.
*Der negative Einfluss (kleineres Nutzsignal) überwiegt hier gegenüber dem positiven  (weniger Störung).  
+
*The negative influence&nbsp; (smaller useful signal)&nbsp; outweighs the positive influence (less noise).
 
{{ML-Fuß}}
 
{{ML-Fuß}}
  
  
  
[[Category:Aufgaben zu Stochastische Signaltheorie|^5.4 Matched-Filter^]]
+
[[Category:Theory of Stochastic Signals: Exercises|^5.4 Matched Filter^]]

Latest revision as of 15:56, 21 February 2022

Given Gaussian pulse

At the input of a reception filter there is

  • a Gaussian pulse   $g(t)$  with amplitude  $g_0$  and equivalent duration $\Delta t_g = 1\hspace{0.08cm} \rm ms$,
$$g(t) = g_0 \cdot {\rm{e}}^{ - {\rm{\pi }}\left( {t/\Delta t_g } \right)^2 } ,$$
  • superimposed by white Gaussian noise with power density $N_0 = 10^{-4}\hspace{0.08cm} \rm V^2 \hspace{-0.1cm}/Hz$.


The pulse energy is  $E_g = 0.01\hspace{0.08cm} \rm V^2 s$.  Let the reception filter be an acausal Gaussian low-pass filter with frequency response

$$H_{\rm E} (f) = {\rm{e}}^{ - {\rm{\pi }}\left( {f/\Delta f_{\rm E} } \right)^2 } .$$

The associated impulse response is thus:

$$h_{\rm E} (t) = \Delta f_{\rm E} \cdot {\rm{e}}^{ - {\rm{\pi }}\left( {\Delta f_{\rm E} \hspace{0.03cm}\cdot \hspace{0.03cm}t} \right)^2 } .$$

The system-theoretical filter bandwidth  $\Delta f_{\rm E}$  should be chosen so that the Gaussian low-pass filter is optimally matched to the input impulse  $g(t)$.  This is then referred to as a:  "matched filter".



Notes:

  • The exercise belongs to the chapter  Matched Filter.
  • Use the following definite integral to solve:
$$\int_0^\infty {{\rm{e}}^{ - a^2 x^2 } {\rm{d}}x = \frac{{\sqrt {\rm{\pi }} }}{2a}} .$$



Questions

1

Calculate the pulse amplitude.

$g_0 \ = \ $

$\ \rm V$

2

What is the maximum S/N ratio at the filter output in  $\rm dB$?

$10 \cdot \lg\ \rho_d(T_\text{D, opt}\hspace{-0.05cm}) \ = \ $

$\ \rm dB$

3

At what filter bandwidth is this S/N ratio achieved?

$\Delta f_\text{E, opt}\ = \ $

$\ \rm kHz$

4

Which of the following statements are true if the filter bandwidth  $\Delta f_{\rm E}$  is smaller than calculated in subtask  (3)

The signal component  $d_{\rm S}(T_\text{D, opt}\hspace{-0.05cm})$  is smaller than with matching.
The noise power  $\sigma_d^2$  is larger than with matching.
The S/N ratio is smaller than calculated in subtask  (2)


Solution

(1)  For the energy of an pulse  $g(t)$  applies in general,  or for this example:

$$E_g = \int_{ - \infty }^{ + \infty } {g^2(t) \hspace{0.1cm}{\rm{d}}t} = g^2_0 \cdot \int_{ - \infty }^{ + \infty } {{\rm{e}}^{ - 2{\rm{\pi }}\left( {t/\Delta t_g } \right)^2 } \hspace{0.1cm}{\rm{d}}t} .$$
  • This equation can be transformed as follows:
$$E_g = 2 \cdot g_0 ^2 \cdot \int_0^\infty {{\rm{e}}^{ - \left( {\sqrt {2 \rm{\pi }} /\Delta t_g } \right)^2 \cdot \hspace{0.05cm} t^2 }\hspace{0.1cm} {\rm{d}}t} .$$
  • With   $a = (2\pi)^{1/2}\cdot 1/\Delta t_g$   and the formula given,  the following relationship holds:
$$E_g = 2 \cdot g_0 ^2 \cdot \frac{{\sqrt {\rm{\pi }} }}{2a} = \sqrt 2 \cdot g_0 ^2 \cdot \Delta t_g .$$
  • Solving this equation for  $g_0$,  the final result is:
$$g_0 = \sqrt {\frac{E_g }{\Delta t_g \cdot \sqrt 2 }} = \sqrt {\frac{{0.01\;{\rm{V}}^{\rm{2}} {\rm{s}}}}{{0.001\;{\rm{s}} \cdot 1.414}}} \hspace{0.15cm}\underline { = 2.659\;{\rm{V}}}.$$


(2)  Assuming a matched filter,  the S/N ratio at the output is:

$$\rho _{d} ( {T_{{\rm{D,}}\,{\rm{opt}}} } ) = \frac{2 \cdot E_g }{N_0 } = \frac{{2 \cdot 10^{ - 2} \;{\rm{V}}^{\rm{2}} {\rm{s}}}}{{10^{ - 4} \;{\rm{V}}^{\rm{2}} {\rm{/Hz}}}} = 200.$$
  • In logarithmic representation,  the following result is obtained:
$$10 \cdot \lg \rho _{d} ( {T_{{\rm{D,}}\,{\rm{opt}}} } ) = 10 \cdot \lg \left( {200} \right) \hspace{0.15cm}\underline {\approx 23\;{\rm{dB}}}.$$


(3)  A comparison between the input pulse and the filter frequency response shows that when  $\Delta f_{\rm E} = 1/\Delta t_g$  is fitted,  it must hold:

$$\Delta f_{{\rm{E,}}\,{\rm{opt}}} \hspace{0.15cm}\underline { = 1\;{\rm{kHz}}}.$$


(4)  Solutions 1 and 3  are correct:

  • A smaller filter bandwidth is favorable with respect to interference,
  • but unfavorable with respect to the useful signal.
  • The negative influence  (smaller useful signal)  outweighs the positive influence (less noise).